Answer:
Step-by-step explanation:
Given:
AB ≅ DC and AC ≅ DB
To Prove:
ΔABC ≅ ΔDCB
Statements Reasons
1). AB ≅ DC and AC ≅ DB 1). Given
2). BC ≅ CB 2). Reflexive property
3). ΔABC ≅ ΔDCB 3). SSS property of congruence
UNIT ISCALING
SCALE DRAWING OF A SCHOOL BUS HAS A SCALE OF 1/2 INCH TO 5 FEET. IF THE
ENGTHS OF THE SCHOOL BUS IS 4 1/2 INCHES ON THE SCALE DRAWING, WHAT IS THE
ACTUAL LENGTH OF THE BUS? EXPLAIN OR SHOW YOUR REASONING
The actual length of the bus is given as follows:
45 feet.
How to obtain the actual length of the bus?The actual length of the bus is obtained applying the proportions in the context of the problem.
The scale drawing is of 0.5 inches to 5 feet, hence the length represented by each inch of drawing is given as follows:
5/0.5 = 10 feet.
The length of the drawing is given as follows:
4.5 inches.
Hence the actual length of the bus is given as follows:
4.5 x 10 = 45 feet.
More can be learned about proportions at https://brainly.com/question/24372153
#SPJ1
Simplify the rational expression
7x/14x^6
The simplified form of the rational expression \((7x)/(14x^6)\) is \(1/(2x^6).\)
To simplify the rational expression \((7x)/(14x^6)\), we can begin by simplifying the numerator and denominator separately.
In the numerator, 7x, we can see that both 7 and x have a common factor of 7. So, we can rewrite the numerator as 7 × x = 7x.
In the denominator, \(14x^6\), we can see that both 14 and x^6 have a common factor of \(2x^6\).
So, we can rewrite the denominator as
\(14 \times x^6 = 2 \times 7 \times x^6 = 2 \times 7x^6.\)
Now, we can simplify the rational expression by canceling out the common factors in the numerator and denominator:
\((7x)/(14x^6) = (7x)/(2 \times 7x^6) \\= (7x)/(2 \times 7 \times x^6) \\= (cross(7x))/(2 \times cross(7) \times x^6) \\= 1/(2x^6)\)
Therefore, the simplified form of the rational expression\((7x)/(14x^6)\) is \(1/(2x^6).\)
for such more question on rational expression
https://brainly.com/question/28832980
#SPJ8
What are the terms a0, a1, a2 and a3 of the sequence {an}, where an equals
a. 2n + n
b. n(n+1)!
c. [n/2]
d. [n/2] + [n/2]
The terms a0, a1, a2, and a3 of the sequence {an} in each case are as follows:
a. a0 = 0, a1 = 3, a2 = 6, a3 = 9
b. a0 = 0, a1 = 2, a2 = 12, a3 = 72
c. a0 = 0, a1 = 0.5, a2 = 1, a3 = 1.5
d. a0 = 0, a1 = 1, a2 = 2, a3 = 3
The sequence {an} is defined asa. 2n + n
a0 = 2*0 + 0 = 0
a1 = 2*1 + 1 = 3
a2 = 2*2 + 2 = 6
a3 = 2*3 + 3 = 9.
b. n(n+1)!
a0 = 0(0+1)! = 0
a1 = 1(1+1)! = 2
a2 = 2*(2+1)! = 12
a3 = 3(3+1)! = 72
c. [n/2]
a0 = [0/2] = 0
a1 = [1/2] = 0.5
a2 = [2/2] = 1
a3 = [3/2] = 1.5
d. [n/2] + [n/2]
a0 = [0/2] + [0/2] = 0
a1 = [1/2] + [1/2] = 1
a2 = [2/2] + [2/2] = 2
a3 = [3/2] + [3/2] = 3
To know more on sequence
https://brainly.com/question/28615767
#SPJ4
Charlie will run at most 35 miles this week. So far, he has run 17 miles. What are the possible numbers of additional miles he will run?
please help can not figere out 9th grade math
by matthew
Step-by-step explanation: We may determine the range of potential further miles Charlie will run by deducting his present mileage from the maximum mileage if he has already run 17 miles and will only cover 35 miles this week.
Maximum mileage - current mileage = potential extra miles.
17 miles minus 35 miles equals 18 miles.
As a result, Charlie may run somewhere between 0 and 18 additional miles.
Learn more about this ans :
https://brainly.com/question/29215649
Answer:
18 possible numbers
Step-by-step explanation:
If Charlie has already run 17 miles and he will run at most 35 miles this week.
We can calculate the possible numbers of additional miles he will run by subtracting the distance he has already run from the maximum distance he can run.
Maximum distance Charlie can run = 35 miles
Distance Charlie has already run = 17 miles
Possible additional miles he will run = Maximum distance - Distance already run
= 35 miles - 17 miles
= 18 miles
Therefore, there are 18 possible numbers of additional miles Charlie will run.
Determine the average rate of change of this function over the interval -2≤x≤4
The rate of change of the function h(x) = 2x on the interval 2 ≤ x ≤ 4 is 6.
The athletic departments at 10 randomly selected U.S. universities were asked by the Equal Employment Opportunity Commission to state what percentage of their nursing scholarships were presently held by women. The responses were 5, 4, 2, 1, 1, 2, 10, 2, 3, 5. what is the midrange
Answer:
5.5
Step-by-step explanation:
Given the set of data
5, 4, 2, 1, 1, 2, 10, 2, 3, 5.
The average of the least and the greatest value is known as the midrange
The formula for calculating the midrange is expressed as shown:
Midrange = (Greatest value + Least value)/2
Given
Greatest value = 10
Least value = 1
Midrange = 10+1/2
Midrange = 11/2
Midrange = 5.5
Hence the midrange of the data is 5.5
Help its urgent i need to get these done
The planes that are parallel in the cube shown would be C. NOR and LMP.
What are parallel planes ?Two planes that are located on a cube and are always opposite and never intersect; they remain continuously at the same distance from each other. A cube consists of three pairs of parallel planes in correspondence with its triplet of opposing faces.
From the given options, the only parallel planes would be NOR and LMP. One of the reasons for this, is that these planes have no point of intersection unlike the planes in the other options.
Find out more on parallel planes at https://brainly.com/question/29329147
#SPJ1
Evaluate. 5^3−(6/48)+10^2 Enter your answer in the box. 100 POINTS!!! PLEASE HELP
Answer:
= 1799 or 224.875
8
Step-by-step explanation:
5³ − (6/48) + 10²
= 125 - (0.125) + 100
= 224.875
or
5³ − (6/48) + 10²
= 125 - 1/8 + 100
= -1/8 + 225
= (225 * 8)/8 - 1/8
= (225 * 8-1) / 8
= 1799 or 224.875
8
Answer:
\(\huge \boxed{\frac{1799}{8} }\)
\(\rule[225]{225}{2}\)
Step-by-step explanation:
\(\displaystyle 5^3-(\frac{6}{48} )+10^2\)
Solving for exponents:
\(\displaystyle 125-(\frac{6}{48} )+100\)
Adding or subtracting:
\(\displaystyle -(\frac{1}{8} )+100+125\)
\(\displaystyle -(\frac{1}{8} )+225\)
\(\displaystyle -(\frac{1}{8} )+\frac{1800}{8}\)
\(\Rightarrow \ \displaystyle \frac{1799}{8}\)
\(\rule[225]{225}{2}\)
How do I solve this question
what is 11/2 times 1/3 as a fraction
Answer:
11/2 times 1/3 is 11/6
Step-by-step explanation:
11/2 times 1/3,
We have to multiply the numerators and denominators,
\((11/2)(1/3) = (11*1)/(2*3) = 11/6\\11/6\)
hence we get 11/6
5) Solve the equation -3(-7 - x) = (x + 2). please help!!
Answer:
x = -19/2
Step-by-step explanation:
the harbormaster wants to place buoys where the rivewr bottom is 20 feet below the surface of the water complete the absolute value equation to find the horizontal distance from the left shore at wich the buoys should be placed
The heights at which the river bottom is 20 feet below the surface of the water is given as follows:
h = 100 m or h = 380 m.
How to obtain the desired heights?The function for this problem is given as follows:
d(h) = 0.2|h - 240| - 48
The variables are given as follows:
d is the depth of the river bottom.h is the horizontal distances.The horizontal distance when the river bottom is 20 feet below the surface water is given by h when d(h) = -20, as follows:
-20 = 0.2|h - 240| - 48
0.2|h - 240| = 28 (applying the commutative property and moving 48 to the other side).
|h - 240| = 28/0.2
|h - 240| = 140.
The absolute value means that there are two possible solutions:
h - 240 = -140.h - 240 = 140.The first solution is then obtained as follows:
h - 240 = -140
h = -140 + 240
h = 100m.
The second solution is then obtained as follows:
h - 240 = 140
h = 140 + 240
h = 380 m.
Missing InformationThe missing text section for the problem is given as follows:
"The bottom of a river makes a V-shape that can be modeled with the absolute value function, d(h) = 0.2|h - 240| - 48, where d is the depth of the river bottom (in feet) and h is the horizontal distance to the left-hand shore (in feet). A ship risks running aground if the bottom of its keel (its lowest point under the water) reaches down to the river bottom."
More can be learned about the absolute value function at https://brainly.com/question/3381225
#SPJ1
Does the expression x^3-1 divided by x^2 -1 simplify to x?
No, the expression (x^3 - 1) / (x^2 - 1) does not simplify to x.
To simplify the expression, let's first factorize both the numerator and denominator.
The numerator can be factorized using the difference of cubes formula: a^3 - b^3 = (a - b)(a^2 + ab + b^2). So, we have (x^3 - 1) = (x - 1)(x^2 + x + 1).
The denominator is a difference of squares: a^2 - b^2 = (a - b)(a + b). Therefore, (x^2 - 1) = (x - 1)(x + 1).
Now, we can simplify the expression by canceling out the common factors in the numerator and denominator:
[(x - 1)(x^2 + x + 1)] / [(x - 1)(x + 1)]
The (x - 1) terms cancel out, leaving us with:
x^2 + x + 1 / (x + 1)
So, the simplified form of the expression is (x^2 + x + 1) / (x + 1), which is not equal to x.
Know more about numerator here:
https://brainly.com/question/1217611
#SPJ8
Simplify.
3(x + 4)
12 x
7 x
3 x + 12
3( x + 4)
Answer:
3(x+4) = 3(x) + 3(4)
=3x+12
Let me know if this helps!
The answer is:
3x + 12Work/explanation:
Simplify by distributing 3:
\(\sf{{3(x+4)}}\)
\(\sf{3\cdot x + 3 \cdot4}\)
\(\sf{3x+12}\)
Hence, the answer is 3x + 12.The property used above is the distributive property.
A fish market bought two swordfish at a rate of $12 per pound. The cost of the larger fish was 3 times as great as the cost of the smaller fish. The total cost of the two fish was $3,648.
How much did each fish weigh?
Answer:
Step-by-step explanation:
The small swordfish costs 912$ and weighs 76 pounds
The large swordfish costs 2736$ and weighs 228 pounds
Mary is going to roll a six-sided die. What is the probability that the die lands on the number 3? Input your answer in fraction form
Answer:
added in the picture
Step-by-step explanation:
added in the picture
Identify the fallacies of relevance committed by the following arguments, giving a brief explanation for your answer. If no fallacy is committed, write "no fallacy"
The First Amendment to the Constitution prevents the government from interfering with the free exercise of religion. The liturgical practice of the Religion of Internal Enlightenment involves human sacrifice. Therefore, it would be wrong for the government to interfere with this religious practice.
The Fallacy mentioned in the arguments is the accident fallacy.
Fallacies are common crimes in logic that will undermine the sense of your argument. Fallacies can be moreover illegitimate arguments or inapplicable points and are frequently linked because they warrant substantiation that supports their claim.
Logical fallacies make an argument weak by using incorrect beliefs ideas, invalid arguments, fallacious arguments, and/or deceptiveness. However, avoid fallacies of study because they produce sins in an argument If you're arguing.
An accident fallacy is a logical fallacy, meaning a logical error that weakens the argument being made, in which a conception is applied to a situation where, in reality, it doesn’t apply. It may be committed due to neglectfulness or because one has the supposition that conceptions will apply to all analogous situations, indeed though there are clear exceptions.
To know more about Fallacy,
brainly.com/question/1970733
brainly.com/question/20939336
#SPJ4
NEED ANWSER ASAP PLS
a horizontal translation only
a horizontal translation and a 180° rotation
a horizontal translation and a glide reflection
a horizontal translation and a reflection across a vertical line
The transformations that map the strip pattern onto itself is a horizontal translation, a reflection across a vertical line, a reflection across a horizontal line, a glide reflection, and a 180°. Option A is the correct option.
Here, we have,
the conditions for mapping the strip transformation pattern onto itself:
The strip pattern can be mapped onto itself in two different ways.
The first way is to translate the strip pattern horizontally and then rotate it by 180° degrees.
The second way is to reflect the strip pattern across a vertical line.
Hence, the transformations that map the strip pattern onto itself can be achieved through a horizontal translation, a reflection across a vertical line, a reflection across a horizontal line, a glide reflection, and a 180°.
Learn more about transformations map on
brainly.com/question/7721186
#SPJ1
complete question:
Which transformations map the strip pattern onto itself?
a horizontal translation, a reflection across a vertical line, a reflection across a horizontal line, a glide reflection, and a 180°
a horizontal translation only
a horizontal translation, a reflection across a horizontal line, and a glide reflection only
a horizontal translation and a 180° rotation only
Please help me answer!
Answer:
B
..................
angie needs to buy 156 candlesbfor a party. each package has 8 candles. how many packages should Angie buy?
20 packages
Step-by-step explanation:
Divide 156 by 8 and you get 19.5. Angie needs 156 candles, so you round 19.5 to 20.
WILL MARK BRAINLYIST
Answer:
the answer would be true
Answer:
FALSE
Step-by-step explanation:
The answer is false because the length between DC is 6 and the length between EB is 8. If the lines were parallel, both lines would have the same length.
1493600÷8 i need full steps
When 1,493,600 is divided by 8 the quotient is 186,700.
To divide 1,493,600 by 8, you can follow these steps:
We have to write down the dividend (1,493,600) and the divisor (8).
Now start with the largest place value in the dividend (the leftmost digit) and perform the division.
Divide 1 by 8. Since 1 is smaller than 8, you move to the next digit.
Bring down the next digit (4) and combine it with the previous quotient (0). This gives you 04.
Divide 4 by 8. Since 4 is smaller than 8, you move to the next digit.
Bring down the next digit (9) and combine it with the previous quotient (0). This gives you 09.
Divide 9 by 8. The quotient is 1, and the remainder is 1.
Bring down the next digit (3) and combine it with the remainder (1). This gives you 13.
Divide 13 by 8. The quotient is 1, and the remainder is 5.
Bring down the next digit (6) and combine it with the remainder (5). This gives you 56.
Divide 56 by 8. The quotient is 7, and there is no remainder.
Bring down the next digit (0) and combine it with the quotient (7). This gives you 70.
Divide 70 by 8. The quotient is 8, and there is no remainder.
There are no more digits to bring down, and the division is complete.
The quotient is the result of the division. In this case, 1,493,600 divided by 8 is equal to 186,700.
Therefore, the result of 1,493,600 ÷ 8 is 186,700.
To learn more on Division click:
https://brainly.com/question/21416852
#SPJ1
The machinery in a cereal plant fills 350 g boxes of cereal. The specifications for the machinery permit for a certain amount of fill tolerance. It is found that the weights of filled cereal boxes are normally distributed with a mean of 350 g and a standard deviation of 4 g. What is the probability that a box of cereal is under filled by 5 g or more?
There is approximately an 89.44% probability that a box of cereal is underfilled by 5 g or more.
To find the probability that a box of cereal is underfilled by 5 g or more, we need to calculate the probability of obtaining a weight measurement below 345 g.
First, we can standardize the problem by using the z-score formula:
z = (x - μ) / σ
Where:
x = the weight value we want to find the probability for (345 g in this case)
μ = the mean weight (350 g)
σ = the standard deviation (4 g)
Substituting the values into the formula:
z = (345 - 350) / 4 = -1.25
Next, we can find the probability associated with this z-score using a standard normal distribution table or a statistical calculator.
The probability of obtaining a z-score less than -1.25 is approximately 0.1056.
However, we are interested in the probability of underfilling by 5 g or more, which means we need to find the complement of this probability.
The probability of underfilling by 5 g or more is 1 - 0.1056 = 0.8944, or approximately 89.44%.
Therefore, there is approximately an 89.44% probability that a box of cereal is underfilled by 5 g or more.
For more questions on probability
https://brainly.com/question/251701
#SPJ8
Using trial and improvement, find the solution between 3 and 4 for the following equation: 2 x 3 − x 2 = 100 Give your answer rounded to 1 DP.
The solution is, After decreasing £16870 by 3% we get, £16363.90.
Here, we have,
A percentage is a number or ratio expressed as a fraction of 100. It is often denoted using the percent sign, "%".
First, find the percentage of £16870 by 3%
%value = 3% * 16870
= (3 / 100) * 16870
= 506.1
Now, just minus with actual value
New value = actual value - %value
We know, actual value = £16870 and %value = 506.1
so, New value = 16870 - 506.1
we get, New value = £16363.90
Therefore, After decreasing £16870 by 3% we get, £16363.90
To read more about Percentage.
brainly.com/question/25896797
#SPJ1
complete question:
Decrease £16870 by 3%
Give your answer rounded to 2 DP.
Let f(x) = √x-5 for x ≥ 5
Find f^-1(2)
Answer: To find the inverse of the function f(x), we need to switch the roles of x and y, so that x becomes the dependent variable and y becomes the independent variable. Then, we need to solve for x in terms of y.
Let y = √x - 5, then x = y^2 + 5.
To find f^-1(2), we need to substitute 2 for y in the equation x = y^2 + 5:
x = 2^2 + 5 = 9
So f^-1(2) = 9.
This means that the inverse of f(x) at 2 is 9. In other words, if f(x) = 2, then x = 9.
Step-by-step explanation:
In the 1960 Olympics, Wilma Rudolph of the United States won the women's 100 meter run in 11.0 seconds. In 1988, Florence Griffith-Joyner, also of
the United States, won with a time of 10.49 seconds, which is still the fastest women's time to this date. Find the percent of decrease in the winning
time. Round to the nearest tenth
% decrease
Hello,
I hope you and your family are doing well!
To find the percent of decrease in the winning time, you can use the following formula:
percent decrease = (old value - new value) / old value * 100%
Plugging in the values from the problem, we get:
percent decrease = (11.0 seconds - 10.49 seconds) / 11.0 seconds * 100% = 0.51 seconds / 11.0 seconds * 100% = 4.636363636363636%
Rounding to the nearest tenth, the percent of the decrease in the winning time is:
4.6%
This means that the winning time decreased by 4.6% between the 1960 Olympics and the 1988 Olympics.
--
Please consider giving this 5 stars and brainliest if you find it helpful.
Happy Holidays!
Help Please I will give 20points
Answer:
10 acres
Step-by-step explanation:
Land for coconuts =
\( \frac{3}{8} = \frac{15}{40} \)
Land for bananas =
\( \frac{4}{5} \times \frac{5}{8} = \frac{20}{40} \\ = \frac{4}{8} \\ = \frac{1}{2} \)
Land left for rice =
\( \frac{40}{40} - \frac{15}{40} - \frac{20}{40} = \frac{5}{40} \\ = \frac{1}{8} \)
Land for bananas =
\( \frac{1}{8} = 2.5 \\ \frac{4}{8} = 2.5 \times 4 \\ = 10\)
Dracula wants to earn more than $142 for mowing yards. He earns $18 per yard. Which inequality shows how many yards Dracula must mow?
Answer:
The inequality that can represent it is;
\(18C>142\)Explanation:
Given that Dracula wants to earn more than $142 for mowing yards.
If he earns $18 per yard.
Let C represent the number of yards he must mow.
The amount Dracula will earn from mowing C yards is;
\(18\times C=18C\)since Dracula wants to earn more than $142, then the inequality that can represent it is;
\(18C>142\)16. A photograph is reduced to 75% of its original size. The photo was originally 96 squaro Inches. What is
the now area after it has boon reduced?
Answer:
mom clock usualli a h i j o
I WILL GIVE BRAINILY PLSSSSS HELPLPPP
Which linear function is increasing at a greater
rate? Explain your reasoning.
Linear Function 1 has an x-intercept of (4, 0) and
a y-intercept at (0, 22)
Linear function 2 includes the points in the table
below
Answer:
Linear function 2
Step-by-step explanation:
If Linear function 1 has an x-intercept of 4,0 and a y-intercept of 0,22
It's a negative equation, Its not increasing Its decreasing.
So it must be Linear function 2